Notation Help: Understanding Summations

  • Thread starter Thread starter holezch
  • Start date Start date
  • Tags Tags
    Notation
Click For Summary
The notation \sum_{i,j} A_{i,j} indicates a double summation where both indices i and j are independent and range over their respective domains, typically the natural numbers. This expression can be interpreted as summing over all pairs (i,j) in the Cartesian product of their ranges. The summation is independent of the order in which i and j are accumulated, as addition is commutative. For example, if both i and j range from 1 to 3, the sum includes all combinations of A_{ij} values, totaling 9 distinct terms. Ultimately, \sum_{i,j} A_{ij} represents the sum of all possible values of A_{ij} across the specified ranges.
holezch
Messages
251
Reaction score
0

Homework Statement



Hi, I see something like this in my book:

\sum_{i,j} A_{i,j}... et c

does this mean that the index values of i and j are both accumulated at the same time? or that i or j gets accumulated first? I'm not sure

thanks
 
Physics news on Phys.org
Generally that really means you have a double summation, that i and j are independent of each other and both range over the natural numbers (or whatever domain makes sense).

You can also think of it as summing over all pairs (i,j) in \mathbb{N}x\mathbb{N}.

Either way, implicitly stated is that the summation is independent of order (since none is given)
 
It is usually shorthand for
\sum_i \sum_j \cdots \sum_n A_{ij\cdots n},
i.e. a nested summation.
 
If, for example, i and j can both range from 1 to 3, that is
A_{11}+ A_{12}+ A_{13}+ A_{21}+ A_{22}+ A_{23}+ A_{31}+ A_{32}+ A_{33}.

That is, A_{ij} has 3(3)= 9 values and this is the sum of all of them. Since addition of numbers is commutative, the order does not matter so the order in which you take i or j does not matter.

More generally, if i ranges from 1 to m and j ranges from 1 to n, A_{ij} can have mn values and \sum_{i, j} A_{ij} is the sum of all of them.
 
Question: A clock's minute hand has length 4 and its hour hand has length 3. What is the distance between the tips at the moment when it is increasing most rapidly?(Putnam Exam Question) Answer: Making assumption that both the hands moves at constant angular velocities, the answer is ## \sqrt{7} .## But don't you think this assumption is somewhat doubtful and wrong?

Similar threads

Replies
5
Views
3K
Replies
0
Views
859
  • · Replies 7 ·
Replies
7
Views
2K
Replies
1
Views
863
  • · Replies 4 ·
Replies
4
Views
1K
  • · Replies 12 ·
Replies
12
Views
2K
Replies
9
Views
2K
  • · Replies 2 ·
Replies
2
Views
3K
  • · Replies 4 ·
Replies
4
Views
1K
  • · Replies 14 ·
Replies
14
Views
3K